Hirdetés

2024. május 1., szerda

Gyorskeresés

Hozzászólások

(#4401) asuspc96


asuspc96
senior tag

Köszönöm a magyarázatokat :)

Jester01 Azért írtam ide mert a fizika bevan halva

(#4402) emiki6


emiki6
veterán

Van egymás mellett 5 ház, mind az 5 különböző színű. A házakban lakik egy-egy személy, mindegyik különböző nemzetiségű. Mindegyik fogyaszt valamilyen italt, dohányárut és tart valamilyen állatot. Egyikük sem fogyaszt ugyanolyan italt, szív ugyanolyan cigarettát, és tart ugyanolyan állatot.

Egyéb információk:
– A brit piros házban lakik.
– A svéd kutyákat tart.
– A dán teát iszik.
– A fehér ház balján a zöld ház van.
– A zöld házban kávét fogyasztanak .
– Az a személy, aki Pall Mallt szív, madarakat tart.
– A sárga ház lakója Dunhillt szív.
– A középső házban lakó tejet iszik.
– A norvég az első házban lakik.
– A Blendet szívó szomszédjában lakó macskát tart.
– A Blue Mastert szívó ember sörözik.
– A lovakat tartó szomszédjában lakó Dunhillt szív.
– A német Price-t szív.
– A norvég a kék ház szomszédja.
– A Blendet szívó szomszédságában vizet isznak.

Kérdés: Ki tart halakat?

Vótmá? Ha nem, sok sikert!

A megoldást leírom a Próba topikba és linkelem. :K

[ Szerkesztve ]

Kívánom neked, hogy mindent megkapj az élettől, hogy rájöjj, nem elég. - - - - - - - - - - - - - - - - - - - - - - - - - - - - - - - - - Quad Era-1, Grado Hemp & Neumann NDH-30 "salesman"

(#4403) emiki6 válasza emiki6 (#4402) üzenetére


emiki6
veterán

Megoldás

Előfordulhat szerintem, hogy több megoldás is van, de nekem ez jött ki.

Kívánom neked, hogy mindent megkapj az élettől, hogy rájöjj, nem elég. - - - - - - - - - - - - - - - - - - - - - - - - - - - - - - - - - Quad Era-1, Grado Hemp & Neumann NDH-30 "salesman"

(#4404) Đusty válasza emiki6 (#4402) üzenetére


Đusty
addikt

Biztos, hogy volt már mert ez egy baromi elterjedt feladvány amit állítólag Einstein talált ki és csak egyjegyű százaléknyi ember tudja megoldani ami mind nem igaz csak ez a reklámja. Ha beírod a keresőbe lesz rengeteg találat szóval egy bizonyos nem túl magas életkor felett mindenki ismeri, "de egy újszülöttnek minden vicc új". De legalább most már meg van itt is. :)

(#4405) emiki6 válasza Đusty (#4404) üzenetére


emiki6
veterán

Lehet már én is találkoztam vele csak elfelejtettem. :DDD

Kívánom neked, hogy mindent megkapj az élettől, hogy rájöjj, nem elég. - - - - - - - - - - - - - - - - - - - - - - - - - - - - - - - - - Quad Era-1, Grado Hemp & Neumann NDH-30 "salesman"

(#4406) #36268800 válasza Jester01 (#4400) üzenetére


#36268800
törölt tag

Szia!

Ne haragudj, de talán az én hozzászólásom elkerülte a figyelmedet: #4397

[ Szerkesztve ]

(#4407) #36268800


#36268800
törölt tag

Ha 1 GHz (10^9 Hz) periódusideje 1/10^9 s, azaz 1 nanoszekundum (10^-9 s),
akkor 2,4 GHz periódusideje miért pikoszekundum (10^-12) időtartamú?
Nem egyszerűen csak 2,4 nanoszekundumról van szó?

(#4408) TDX


TDX
tag

A herz azt adja meg, hogy egy másodperc alatt mennyit fordul. A periódusidő hogy egyet mennyi idő alatt fordul meg. Tehát a periódusidő az a frekvencia reciproka.
Ebből egyszerű reciprokképzéssel: 2,4 GHz-es frekvenciánál a periódusidő 1/(2,4*10^9)=4,166*10^-10s.

Már vége az Én hozzászólásomnak? Mi lesz ez után velünk?!?!

(#4409) #36268800 válasza TDX (#4408) üzenetére


#36268800
törölt tag

Oké, ez így tiszta! Csak az nem, hogy akkor ez miért pikoszekundum és nem nanoszekundum?
ns = 10^(-9)
ps = 10^(-12)

és itt 10^(-10)-ről van szó. Ezek szerint a [10^(-9), 10^(-12)] intervallum jelenti a pikoszekundumot?
Tehát értelemszerűen, mivel a negatív számok halmazáról van szó, 'a kisebb a nagyobb'?

(#4410) TDX válasza #36268800 (#4409) üzenetére


TDX
tag

Én ha nem értek valamit, akkor beütöm a gugliba, és az általában kidobja. Jó, nem mond mindig igazat,de a tényszerű adatokban ritkán téved, a mindigelsőtalálat wikipédia, most sincs ez máshogy. a 10^-9 és 10^-12 között nincs az SI-ben nincs nagyságrendi preffixum, tehát ha nem s-ben (normál alakban) adod meg (4,16*10^-10 s), akkor a nála egyel kisebb prefixum a pikosekundum, ami 10^-12. Tehát a periódusideje 4,16*10^-10s=4,16*10^-1ns=4,16*10^2ps.

Már vége az Én hozzászólásomnak? Mi lesz ez után velünk?!?!

(#4411) DrojDtroll


DrojDtroll
addikt

Sziasztok!

Adott a következő feladat:

Mivel egyenlő:

ha

Én ezt úgy oldottam meg, hogy a kiszámltam az egyenletből x értékét és visszahelyetesítettem. Van erre valami jobb megoldás, vagy levezetés, is vagy jó úgy ahogy csináltam?

(#4412) Cucuska2 válasza DrojDtroll (#4411) üzenetére


Cucuska2
őstag

Az alsót négyzetre emeled, és kijön, hogy a felső 47.

Rock and stone, to the bone! Leave no dwarf behind!

(#4413) DrojDtroll válasza Cucuska2 (#4412) üzenetére


DrojDtroll
addikt

Köszi :)

(#4414) nagybá válasza Cucuska2 (#4412) üzenetére


nagybá
senior tag

Oké oké, de akkor a 2x/X-el mi lesz? :-| Vagy én vagyok ennyire sügér? :D x és X nem ugyanaz nem?

(#4415) Apollo17hu válasza nagybá (#4414) üzenetére


Apollo17hu
őstag

Valószínűleg ugyanaz, ezért x/X = 1, vagyis 2x/X = 2, ahonnan 49 - 2 = 47.

(#4416) DrojDtroll válasza Apollo17hu (#4415) üzenetére


DrojDtroll
addikt

A két x valóban ugyan az, nem tudom a képlet szerkesztő miért így jelenítettem meg.

(#4417) nagybá


nagybá
senior tag

Ja hát így más.

(#4418) bundli


bundli
tag

Üdv.!

Van ötletetek, hogy hogyan lehet egy parabola egyenletét megkapni 2 pontból és a fókuszponjából?

Tehát 2 pontot lerakok egymás mellé mondjuk, és középre felé egy fókuszpontot. Ebből kéne kiszámolni a parabola egyenletét. Természetesen a 2 pont szimmetrikus a fókuszpontra.

(#4419) Jester01 válasza bundli (#4418) üzenetére


Jester01
veterán

Ha a parabola egyenletét y = a(x - h)^2 + k alakban vesszük akkor a fókusz koordinátái (h, k + 1/(4a)). Innen h egyből adódik, k és a pedig egy lineáris egyenletrendszerből (elég az egyik pont hozzá).

Jester

(#4420) bundli válasza Jester01 (#4419) üzenetére


bundli
tag

Köszönöm szépen a segítséged, ez valóban működik! :R

(#4421) bundli


bundli
tag

Jönnék akkor mégegy kérdéssel.

Van ez az egyeneletem:

Kifejeztem Y-ra egy matematikai programmal (Matlab), de kettő egyenletet is adott. Ki is próbáltam őket, egyszer vagy az egyikkel adott helyes megoldást, máskor a másikkal. Mitől fordulhatott ez elő? Valaki meg tudja magyarázni, hogy matematikailag melyiket kéne használni és mikor?

(#4422) Cucuska2 válasza bundli (#4421) üzenetére


Cucuska2
őstag

Mert mi a feladat? u, v paraméter? x, y változó? a, b is adott? Miért akarod kifejezni y-ra? Nagyon rusnya lesz az.

Rock and stone, to the bone! Leave no dwarf behind!

(#4423) bundli válasza Cucuska2 (#4422) üzenetére


bundli
tag

a,b,c,u,v konstansok, X és Y ismeretlen. Y-ra kéne kifejezni. Ez egy parabola egyenlet egyébként. Ennél szebb alakja nincsen.

(#4424) gygabor88 válasza bundli (#4421) üzenetére


gygabor88
tag

Ötlet: Az y^2 és y együtthatói alapján szét lehet bontani több esetre. Pl a=0 esetben eleve kiesik az y^2, de ekkor még mindig vizsgálni kell y együtthatójának értékét. A v + (c/b) = 0 esetben x = u egyenes lesz a megoldás, míg v + (c/b) != 0 esetben parabolát kapsz. Persze itt b != 0 és a = 0 volt. Az a != 0 esetben már x-re is meg kell vizsgálni pár dolgot, nem csináltam végig és esélyes, hogy fentebb is elszámolhattam így késő este. :)

[ Szerkesztve ]

(#4425) Jester01 válasza bundli (#4423) üzenetére


Jester01
veterán

Mind a két megoldás helyes, mivel a parabola nem csak függőleges lehet.
Ott van rögtön a négyzetgyök ami teljes pompájában ugye az x tengelyre szimmetrikus.

Jester

(#4426) bundli válasza gygabor88 (#4424) üzenetére


bundli
tag

Igen, ez valószínű, hogy a különböző paraméterek értékétől függ, hogy éppen melyik egyenlet az ami fog kelleni nekem. Namost. Hogyan tudom eldönteni mondjuk kirajzoláskor, hogy nekem melyik implicit egyenletbe kell behelyettesíteni?

Ha jól értem, akkor azt az együtthatót kéne vizsgálnom, ami eldönti, hogy most a kettő közül melyik egyenlet lesz a jó. Már csak azt kéne kitalálni hogy hogyan találom meg azt. És mi van ha több is beleszól a dologba egyszerre. :F

[ Szerkesztve ]

(#4427) gygabor88 válasza bundli (#4426) üzenetére


gygabor88
tag

Kirajzoláskor (x-tengely mentén haladva) mindkettőbe vissza kell helyettesítened x-et, mert adott x-hez 0, 1 vagy 2 valós y is hozzá lehet rendelve és minden így kapott (x, y) pár megoldása lesz az eredeti egyenletnek.

(#4428) bundli válasza gygabor88 (#4427) üzenetére


bundli
tag

Igen, mondjuk ez kirajzoláskor működött is. Viszont miután lederiváltam X szerint a két implicit egyenletet, akkor miután érintőt húztam ezek segítségével a parabola egy pontjába, akkor két egyenesem érintőm lett. Az egyik általában merőleges a másikra. A két egyenes közül az egyik mindig jó, de nem jöttem rá, hogy mikor melyiket kéne kirajzolnom a helyes eredményhez.

Természetesen a parabola helyzettől függ, hogy mikor melyik egyenlet működik.

[ Szerkesztve ]

(#4429) gygabor88 válasza bundli (#4428) üzenetére


gygabor88
tag

A MatLab kiadott neked két x-től függő függvényt, amiket kirajzolva megkaptad az eredeti egyenlet megoldáshalmazát. Ha ezután egy (x, y) pontba érintőt akarsz húzni, akkor tudnod kell, hogy ez a megoldáspár melyik függvényhez (egyenlethez) tartozott. Ezt (x, y) visszahelyettesítéssel tudod ellenőrizni mindkét esetben és amelyikre jó az (x, y) pár, annak a függvénynek a deriváltjával számított meredekséggel tudsz érintőt húzni az adott pontba.

(#4430) bundli válasza gygabor88 (#4429) üzenetére


bundli
tag

Ezt (x, y) visszahelyettesítéssel tudod ellenőrizni mindkét esetben és amelyikre jó az (x, y) pár,

Pontosan. De ezt hogyan tudom kitalálni ránézésre, hogy melyik a jó? A behelyettesítés értékéből? Az x,y párból?

(#4431) gygabor88 válasza bundli (#4430) üzenetére


gygabor88
tag

Az x, y párból. Ugyanis ha megvan ez az (x, y) pár, akkor tudod, hogy adott x-hez milyen y-nak kell tartozni az adott pontban. Tehát x-et visszaírod mindkét függvénybe és amelyik az elvárt y-t adja vissza, azzal a függvénnyel dolgozhatsz tovább. Ez ránézésre nem biztos, hogy eldönthető, a fentebb leírt módon kell egy keveset számolni hozzá.

[ Szerkesztve ]

(#4432) bundli válasza gygabor88 (#4431) üzenetére


bundli
tag

Az érintő egyenletét a következőképpen számolom.

A derivált Y-ok egyenletébe behelyettesítem az x0-t, ez megad két meredekséget. (m1 vagy m2)

Majd ebbből:

Y1=m1*(x*x0)+y0
Y2=m2*(x*x0)+y0

Ez adja meg magát az érintők egyenleteit.

Tehát az egyetlen dolog amivel játszani kell az m, mert abból van kettő.

Namost. Én látom az m-ek értékeit, ezek egymással ellentétes előjelűek. Próbáltam úgy, hogy mindig a negatívat választottam jónak, sajnos ez nem segített.

Te hogyan gondoltam (x0,y0)-ból megmondani, hogy melyik m-et kéne válaszani?

(Elnézést, hogy értetlenkedek, de nagyon fontos lenne nekem és ahogy tudom viszonozni is fogom a segítséget!)

(#4433) gygabor88 válasza bundli (#4432) üzenetére


gygabor88
tag

Deriválás előtt kell visszahelyettesíteni.
Nézzük azt az esetet, amikor minden konstansod 1. A csatolt képen látszik az Out[14] sorban a két hozzárendelés, ami megadja az (x, y) megoldáspárokat az egyenlethez. Ez a két függvény alul lett kirajzolva.

Most tegyük fel, hogy tudod, az (x0, y0) = (4, 7 - 3sqrt(6)) benne van ebben a megoldáshalmazban és szeretnél érintőt húzni ebbe a pontba. Fogod az x0 = 4-et és beírod a két kifejezésbe, amit az Out[14] sorban látsz. Az első azt adja, hogy y -> 7 - 3sqrt(6) a második pedig, hogy y -> 7 + 3sqrt(6). De te tudod, hogy az (x0, y0) pontodban az y0 értéke 7 - 3sqrt(6), ez az érték pedig az első visszahelyettesítésből kapott értékkel egyezik meg. Innentől már csak ezt az első függvényt deriválod és ebből számolsz meredekséget.

[ Szerkesztve ]

(#4434) bundli válasza gygabor88 (#4433) üzenetére


bundli
tag

Na, pár órán át szenvedtem ezzel. Egy baj van evvel a megoldással, hogy az a pont ahova az érintőt húzom, az nem 100%-ig pontos, tehát nincs rajta a parabolán, csak nagyon közel van, mivel közelítéssel számoltam azt.

Így az általad leír módszer nem működik, mert néha negatív a diszkrimináns vagy pedig 0 van a nevezőben, mikor behelyettesítek.

Olyan megoldást kéne találni ami az a,b,c,p,q együtthatókkal számol.

Pl.: Ha a,b és p egyszerre nagyobb mint 0, akkor Y1, egyébként meg Y2-t kell használni.

Csak gondolom ez nem igazán játszik...

(#4435) bundli válasza bundli (#4434) üzenetére


bundli
tag

Az előző problémát sikerülne kiváltani a következő probléma megoldásával: katt

Hátha valaki rárjön, hogy mi is a baj. :)

(#4436) Jester01 válasza bundli (#4435) üzenetére


Jester01
veterán

A rajzot nem értem, mi a Q egyáltalán.
De ez amúgy triviálisan adódik onnan ha a P1 ,P2 pontokból meghatározod az egyenest majd annak a P3 x koordinátájánál vett értékét összehasonlítod a P3 y koordinátájával.

Jester

(#4437) bundli válasza Jester01 (#4436) üzenetére


bundli
tag

Igen, ez lett a megoldás, közben én is rátjöttem. Szokásos módon túlbonyolítottam ezt is... :R

(#4438) emiki6


emiki6
veterán

Ma megálapítottam, hogy 2-t és 3-at kivéve minden prímszám 6 többszörösénél 1-el nagyobb vagy kisebb ( n•6-1 vagy n•6+1 ), persze visszafele nem igaz. Elsőre meglepett, de jobban belegondolva elég logikus, mégis érdekes felvetés a részemről. :D

Van már ilyen tétel? :D

[ Szerkesztve ]

Kívánom neked, hogy mindent megkapj az élettől, hogy rájöjj, nem elég. - - - - - - - - - - - - - - - - - - - - - - - - - - - - - - - - - Quad Era-1, Grado Hemp & Neumann NDH-30 "salesman"

(#4439) Jester01 válasza emiki6 (#4438) üzenetére


Jester01
veterán

Tulajdonképpen már az is ilyen tétel, hogy a 2-t kivéve minden prímszám 2n+1 vagy 2n-1 alakú (persze ez a kettő ugyanaz :) )

Jester

(#4440) Apollo17hu válasza emiki6 (#4438) üzenetére


Apollo17hu
őstag

...mivel 6n-1 és 6n+1 lefedi az összes páratlan számot - kivéve a 3-mal oszthatóakat

(#4441) emiki6 válasza Apollo17hu (#4440) üzenetére


emiki6
veterán

Azért logikus :D

Kívánom neked, hogy mindent megkapj az élettől, hogy rájöjj, nem elég. - - - - - - - - - - - - - - - - - - - - - - - - - - - - - - - - - Quad Era-1, Grado Hemp & Neumann NDH-30 "salesman"

(#4442) moha21


moha21
addikt

Sziasztok!

Házi feladatom megoldásában kérnék segítséget!

Az egyik egy "egyszerű" átalakítás, a másik egy deriválás. Illetve örülnék, ha megmondanátok kb. jó -e a többi feladat amit megcsináltam, illetve ha nem mit rontottam el. Munka mellett alig van időm a sulira, ráadásul a matek annyira nem is az erősségem. :B Ettől független magam szenvedtem össze idáig a házikat.

kép 1

kép2

Az első képen a e ^ 3 ln 4 érdekelne és a lap alján a negyedik gyök alatt lévő függvény deriváltja.

[ Szerkesztve ]

Nem az az igazi férfi aki minden nőt meghódít, hanem aki ismeri a nagyfeszültségű földkábelek szigetelésének technikáját.

(#4443) Cucuska2 válasza moha21 (#4442) üzenetére


Cucuska2
őstag

e^(3*ln4) = ((e^ln(4))^3 = 4^3 = 64.
A lap alján lévő pedig sima láncszabállyal megy, külső fv belső helyen szorozva a belső deriváltja, párszor egymás után alkalmazva. Atomrusnya lesz, de nincs mese, végig kell szenvedni.

Rock and stone, to the bone! Leave no dwarf behind!

(#4444) moha21 válasza Cucuska2 (#4443) üzenetére


moha21
addikt

Szia!
Köszönöm a választ!

A deriválásnál a sorrend nem megy. Illetve a hatványozással nem tudok mit kezdeni. Vagy első körben aszerint kellene deriválni és a zárójel belseje maradna eredeti, utána meg a többi függvény szerint deriválni?

Nem az az igazi férfi aki minden nőt meghódít, hanem aki ismeri a nagyfeszültségű földkábelek szigetelésének technikáját.

(#4445) Cucuska2 válasza moha21 (#4444) üzenetére


Cucuska2
őstag

Van az a hosszú valami a zárójel alatt, legyen b(x). És ez van negyedik gyök alatt, és a reciproka van véve. Akkor te a b(x)^(-1/4)-t deriválod először, azaz -1/4*b(x)^(-5/4)*b'(x), és akkor b'(x) pedig már egy összeg, és haladsz minden kifejezésnek a hasa felé.

[ Szerkesztve ]

Rock and stone, to the bone! Leave no dwarf behind!

(#4446) pomorski válasza petymeg (#4032) üzenetére


pomorski
őstag

Maple, Mathematica, MATLAB

(#4447) pomorski


pomorski
őstag

Nagyon szuper ez a topic, kár hogy sajnos még csak ma vettem észre... Úgyhogy segítségeteket is kérném... Ha valaki tud segíteni, akkor PM-et írjon légyszi!

http://kepfeltoltes.hu/view/151013/segitseg_www.kepfeltoltes.hu_.jpg

Ja, bocs, hogy máshogy nem tudtam ide beilleszteni a képet, csak így, de mindig azt a hibaüzenetet kaptam, hogy a "A megadott kép nem megfelelő (max. 8 Mpx, 3 MB)!".

(#4448) pomorski válasza emiki6 (#4438) üzenetére


pomorski
őstag

Csak azért néztem be, hogy reagált-e valaki a kérdésemre, de mivel senki, akkor én reagálok a máséra...

Van egy tétel számelméletből, mely szerint: Bármely háromnál nagyobb prímszám felírható (6k\pm 1) alakban. Itt a \pm jelöli a plusz-mínuszt (LaTeX-es jelölés). A tételt megtalálod pl. Freud-Gyarmati: Számelmélet c. könyvében.

[ Szerkesztve ]

(#4449) Cucuska2 válasza pomorski (#4447) üzenetére


Cucuska2
őstag

Az a baj, hogy én a matematikusit most kezdtem, más szakon pedig nem kellett ilyen matematika még, így nem tudok okosat mondani. :(

Rock and stone, to the bone! Leave no dwarf behind!

(#4450) pomorski válasza Cucuska2 (#4449) üzenetére


pomorski
őstag

Matematikust?! Nagyszerű!!! Hol, melyik egyetemen?

Copyright © 2000-2024 PROHARDVER Informatikai Kft.